Arccotangent Sum

Geometry Level 4

Evaluate cot ( cot 1 ( 7 ) + cot 1 ( 13 ) + cot 1 ( 21 ) + cot 1 ( 31 ) ) \cot{(\cot^{-1}{(7)}+\cot^{-1}{(13)}+\cot^{-1}{(21)}+\cot^{-1}{(31)})}


The answer is 3.25.

This section requires Javascript.
You are seeing this because something didn't load right. We suggest you, (a) try refreshing the page, (b) enabling javascript if it is disabled on your browser and, finally, (c) loading the non-javascript version of this page . We're sorry about the hassle.

4 solutions

Using the arctan sum formula, we have that

cot 1 ( 7 ) + cot 1 ( 13 ) = tan 1 ( 1 7 ) + tan 1 ( 1 13 ) = tan 1 ( 1 7 + 1 13 1 1 7 1 13 ) = tan 1 ( 2 9 ) . \cot^{-1}(7) + \cot^{-1}(13) = \tan^{-1}(\frac{1}{7}) + \tan^{-1}(\frac{1}{13}) = \tan^{-1}\left(\dfrac{\frac{1}{7} + \frac{1}{13}}{1 - \frac{1}{7}*\frac{1}{13}}\right) = \tan^{-1}(\frac{2}{9}).

Similarly, cot 1 ( 21 ) + cot 1 ( 31 ) = tan 1 ( 1 21 + 1 31 1 1 21 1 31 ) = tan 1 ( 2 25 ) . \cot^{-1}{(21)} + \cot^{-1}{(31)} = \tan^{-1}\left(\dfrac{\frac{1}{21} + \frac{1}{31}}{1 - \frac{1}{21}*\frac{1}{31}}\right) = \tan^{-1}(\frac{2}{25}).

Since tan 1 ( 2 9 ) + tan 1 ( 2 25 ) = tan 1 ( 2 9 + 2 25 1 2 9 2 25 ) = tan 1 ( 68 225 ) = tan 1 ( 4 13 ) = cot 1 ( 13 4 ) , \tan^{-1}(\frac{2}{9}) + \tan^{-1}(\frac{2}{25}) = \tan^{-1}\left(\dfrac{\frac{2}{9} + \frac{2}{25}}{1 - \frac{2}{9}*\frac{2}{25}}\right) = \tan^{-1}(\frac{68}{225}) = \tan^{-1}(\frac{4}{13}) = \cot^{-1}(\frac{13}{4}),

the given expression comes out to cot ( cot 1 ( 13 4 ) ) = 13 4 = 3.25 . \cot(\cot^{-1}(\frac{13}{4})) = \dfrac{13}{4} = \boxed{3.25}.

More easy way is to generalising the sum,

S = cot ( 2 n cot 1 ( n 2 + n + 1 ) ) [ n = 5 ] S = \cot \left(\displaystyle\sum_2^n \cot^{-1}( n^2 + n + 1 )\right)\quad [n=5 ] S = cot ( 2 n tan 1 ( n + 1 n 1 + n ( n + 1 ) ) ) S = \cot\left(\displaystyle\sum_2^n \tan^{-1} \left(\dfrac{ n + 1 - n}{1 + n(n+1)}\right)\right)
Now, rest is your's...

Akhil Bansal - 5 years, 8 months ago

Log in to reply

Great observation, Akhil. I was wondering why those specific numbers were chosen. So following your lead, we now have that

n = 2 5 tan 1 ( n + 1 n 1 + n ( n + 1 ) ) = n = 2 5 ( tan 1 ( n + 1 ) tan 1 ( n ) ) , \displaystyle\sum_{n=2}^{5} \tan^{-1}\left(\dfrac{n + 1 - n}{1 + n(n + 1)}\right) = \sum_{n=2}^{5} (\tan^{-1}(n + 1) - \tan^{-1}(n)),

which telescopes to tan 1 ( 6 ) tan 1 ( 2 ) = tan 1 ( 6 2 1 + 6 2 ) = tan 1 ( 4 13 ) = cot 1 ( 13 4 ) , \tan^{-1}(6) - \tan^{-1}(2) = \tan^{-1}\left(\dfrac{6 - 2}{1 + 6*2}\right) = \tan^{-1}\left(\dfrac{4}{13}\right) = \cot^{-1}\left(\dfrac{13}{4}\right),

and so S = cot ( cot 1 ( 13 4 ) ) = 13 4 . S = \cot(\cot^{-1}\left(\dfrac{13}{4}\right)) = \dfrac{13}{4}.

Brian Charlesworth - 5 years, 8 months ago

S = cot ( arctan ( 1 7 ) + arctan ( 1 13 ) + arctan ( 1 21 ) + arctan ( 1 31 ) ) S=\cot\left(\arctan\left(\dfrac{1}{7}\right)+\arctan\left(\dfrac{1}{13}\right)+\arctan\left(\dfrac{1}{21}\right)+\arctan\left(\dfrac{1}{31}\right)\right)

Let a = 7 + i a=7+i , b = 13 + i b=13+i , c = 21 + i c=21+i and d = 31 + i d=31+i . Then:

S = cot ( arg ( a ) + arg ( b ) + arg ( c ) + arg ( d ) ) ) S=\cot(\arg(a)+\arg(b)+\arg(c)+\arg(d)))

S = cot ( arg ( a b c d ) S=\cot(\arg(abcd)

S = cot ( arg ( ( 7 + i ) ( 13 + i ) ( 21 + i ) ( 31 + i ) ) ) S=\cot(\arg((7+i)(13+i)(21+i)(31+i)))

S = cot ( arg ( ( 10 ) ( 9 + 2 i ) ( 26 ) ( 25 + 2 i ) ) ) S=\cot(\arg((10)(9+2i)(26)(25+2i)))

We can ignore the 10 10 and the 26 26 , since they don't affect the argument:

S = cot ( arg ( 221 + 68 i ) ) S=\cot(\arg(221+68i))

S = 221 68 = 13 4 S=\dfrac{221}{68}=\boxed{\dfrac{13}{4}}

Yay! I like the usage of complex numbers to find tangent values.

Calvin Lin Staff - 5 years, 8 months ago
Joe Potillor
Dec 16, 2016

Arjen Vreugdenhil
Oct 18, 2015

I used similar methods as others here, but first derived cot α = x , cot β = y cot ( α + β ) = x y 1 x + y ; \cot \alpha = x, \cot\beta = y\ \Longrightarrow\ \cot(\alpha+\beta) = \frac{xy-1}{x+y}; or, with fractions, cot α = x u , cot β = y v cot ( α + β ) = x y u v x v + y u . \cot \alpha = \frac x u, \cot\beta = \frac y v\ \Longrightarrow\ \cot(\alpha+\beta) = \frac{xy-uv}{xv+yu}.

Applying this to the given values in pairs, cot ( cot 1 ( 7 ) + cot 1 ( 13 ) ) = 7 13 1 7 + 13 = 90 20 = 9 2 ; \cot(\cot^{-1}(7) + \cot^{-1}(13)) = \frac{7\cdot 13-1}{7+13} = \frac{90}{20} = \frac 9 2; cot ( cot 1 ( 21 ) + cot 1 ( 31 ) ) = 21 31 1 21 + 31 = 650 52 = 25 2 ; \cot(\cot^{-1}(21) + \cot^{-1}(31)) = \frac{21\cdot 31-1}{21+31} = \frac{650}{52} = \frac {25}2; cot ( cot 1 ( 9 2 ) + cot 1 ( 25 2 ) ) = 9 25 2 2 9 2 + 25 2 = 221 68 = 13 4 = 3.25 . \cot\left(\cot^{-1}\left(\frac 92\right) +\cot^{-1}\left(\frac {25}2\right)\right) = \frac{9\cdot 25-2\cdot 2}{9\cdot 2+25\cdot 2} = \frac{221}{68} = \frac {13}4 = \boxed{3.25}.

0 pending reports

×

Problem Loading...

Note Loading...

Set Loading...